Ngọc Nguyễn

giúp mình vs mình tick và theo dõi cho

YangSu
29 tháng 6 2023 lúc 12:49

\(a.x^3+12x^2+48x+64=x^3+3.4x^2+3.4^2x+4^3=\left(x+4\right)^3\)

Thay \(x=6\) vào \(\left(x+4\right)^3=\left(6+4\right)^3=10^3=1000\)

\(b,B=x^3-6x^2+12x-8=\left(x-2\right)^3\)

Thay \(x=22\) vào \(\left(x-2\right)^3=\left(22-2\right)^3=20^3=8000\)

\(c,C=x^3+9x^2+27x+27=x^3+3.3x^2+3.3^2x+3^3=\left(x+3\right)^3\)

Thay \(x=-103\) vào \(\left(x+3\right)^3=\left(-103+3\right)^3=\left(-100\right)^3=-1000000\)

\(d,D=x^3-15x^2+75x-125=x^3-3.5x^2+5^2.3x-5^3=(x-5)^3\)

Thay \(x=25\) vào \(\left(x-5\right)^3=\left(25-5\right)^3=20^3=8000\)

Bình luận (0)
HT.Phong (9A5)
29 tháng 6 2023 lúc 12:51

a) \(A=x^3+12x^2+48x+64\)

\(=x^3+3\cdot4\cdot x^2+3\cdot4^2\cdot x+4^3\)

\(=\left(x+4\right)^3\)

Thay \(x=6\) vào biểu thức A ta có:

\(\left(6+4\right)^3=10^3=1000\)

Vậy: ...

b) \(B=x^3-6x^2+12x-8\)

\(=x^3-3\cdot2\cdot x^2+3\cdot2^2\cdot x-2^3\)

\(=\left(x-2\right)^3\)

Thay \(x=22\) vào biểu thức B ta có:

\(\left(22-2\right)^3=20^3=8000\)

Vậy: ...

c) \(C=x^3+9x^2+27x+27\)

\(=x^3+3\cdot3\cdot x^2+3\cdot3^2\cdot x+3^3\)

\(=\left(x+3\right)^3\)

Thay \(x=-103\) vào biểu thức C ta được:

\(\left(-103+3\right)^3=\left(-100\right)^3=-1000000\)

Vậy: ...

d) \(D=x^3-15x^2+75x-125\)

\(=x^3-3\cdot5\cdot x^2+3\cdot5^2\cdot x-5^3\)

\(=\left(x-5\right)^3\)

Thay \(x=25\) vào biểu thức D ta được:

\(\left(25-5\right)^3=20^3=8000\)

Vậy: ...

Bình luận (0)
Nguyễn Lê Phước Thịnh
29 tháng 6 2023 lúc 12:43

a: A=(x+4)^3

Thay x=6 vào A, ta được:

A=(6+4)^3=10^3=1000

b: B=(x-2)^3

Khi x=22 thì B=(22-2)^3=8000

c: C=(x+3)^3

Khi x=-103 thì C=(-103+3)^3=(-100)^3=-1000000

d: D=(x-5)^3

Khi x=25 thì

D=(25-5)^3=20^3=8000

Bình luận (0)

Các câu hỏi tương tự
Ngọc Nguyễn
Xem chi tiết
Ngọc Nguyễn
Xem chi tiết
Ngọc Nguyễn
Xem chi tiết
Ngọc Nguyễn
Xem chi tiết
Ngọc Nguyễn
Xem chi tiết
Ngọc Nguyễn
Xem chi tiết
Ngọc Nguyễn
Xem chi tiết
Ngọc Nguyễn
Xem chi tiết
Ngọc Nguyễn
Xem chi tiết